Difference between revisions of "2018 AMC 10A Problems/Problem 7"

(Created page with "For how many (not necessarily positive) integer values of <math>n</math> is the value of <math>4000\cdot \left(\tfrac{2}{5}\right)^n</math> an integer? <math> \textbf{(A) }3...")
 
Line 8: Line 8:
 
\textbf{(E) }9 \qquad
 
\textbf{(E) }9 \qquad
 
</math>
 
</math>
 +
 +
== See Also ==
 +
 +
{{AMC10 box|year=2018|ab=A|num-b=6|num-a=8}}
 +
{{MAA Notice}}

Revision as of 15:56, 8 February 2018

For how many (not necessarily positive) integer values of $n$ is the value of $4000\cdot \left(\tfrac{2}{5}\right)^n$ an integer?

$\textbf{(A) }3 \qquad \textbf{(B) }4 \qquad \textbf{(C) }6 \qquad \textbf{(D) }8 \qquad \textbf{(E) }9 \qquad$

See Also

2018 AMC 10A (ProblemsAnswer KeyResources)
Preceded by
Problem 6
Followed by
Problem 8
1 2 3 4 5 6 7 8 9 10 11 12 13 14 15 16 17 18 19 20 21 22 23 24 25
All AMC 10 Problems and Solutions

The problems on this page are copyrighted by the Mathematical Association of America's American Mathematics Competitions. AMC logo.png